Đến nội dung

supermember nội dung

Có 113 mục bởi supermember (Tìm giới hạn từ 07-06-2020)



Sắp theo                Sắp xếp  

#729845 lim$n.\sqrt[3]{u_n}$

Đã gửi bởi supermember on 21-08-2021 - 11:42 trong Dãy số - Giới hạn

Ohh, lâu lắm mới thấy 1 ĐHV Olympiad tiếp tục con đường học toán ở bậc đại học.
Chúc em nhiều thành công & sẽ đóng góp lâu dài cho VMF nhé.




#729881 Tính $P(0)$

Đã gửi bởi supermember on 23-08-2021 - 10:11 trong Đa thức

Đã tìm ra 1 tam thức bậc 2 thỏa yêu cầu bài toán: $ P(x) = 14(x-1)^2 +2$

 

Còn một công đoạn nữa là chứng minh đây là đa thức duy nhất thỏa mãn yêu cầu bài toán.




#729939 $\left\{\begin{align*}6(x+y)(xy+\frac{1}{xy}+2)=(2x^...

Đã gửi bởi supermember on 27-08-2021 - 20:26 trong Phương trình - Hệ phương trình - Bất phương trình

Mọi người giúp mình bài này với ạ: ( Hình như chọn đt KHTN năm nào ý)

Giải hệ phương trình:

$\left\{\begin{matrix} 6(x+y)(xy+\frac{1}{xy}+2)=(2x^2+3y^2)(1+\frac{1}{xy}) & \\ 29(xy+\frac{1}{xy})+62=(9x+13y)(1+\frac{1}{xy}) & \end{matrix}\right.$

 

Bài này làm thế này:

 

Điều kiện : $ x; y $ khác $0$

 

Phương trình đầu tiên sẽ tương đương với: $ 6(x+y)(xy+1)^2 = (2x^2+3y^2)(xy+1)$

 

Tức là có 2 trường hợp có thể xảy ra:

 

Trường hợp 1: $xy +1 =0$

 

Trường hợp này ta rất nhanh chóng giải ra vô nhiệm do phương trình thứ $2$ của hệ tương dương với: $ 29(xy+1)^2 + 4xy = (9x+13y)(1+xy)  \ (*)$

 

Tức là: $-4 = 0$ (vô lý)

 

Trường hợp 2: $6(xy +1)(x+y) = 2x^2+3y^2$

 

Tương đương với: $ xy+1 = \frac{2x^2+3y^2}{6(x+y)}$

 

Thay vào $(*)$ ta có được : $ \frac{29(2x^2+3y^2)^2}{36(x+y)^2} + 4xy = \frac{(2x^2+3y^2)(9x+13y)}{6(x+y)}$

 

Tức là cần giải phương trình :$ 29(2x^2+3y^2)^2 + 144xy(x+y)^2 = 6(x+y)(9x+13y)(2x^2+3y^2)$

 

Phương trình này tương đương với: $ 8x^4  - 120x^3 y + 318 x^2 y^2 - 252 xy^3 + 27 y^4 =0$

 

Đến đây thì không còn khó nữa rồi do phương trình $ 8t^4  - 120t^3 + 318 t^2 - 252 t + 27 =0$ có thể viết được dưới dạng:

 

$ (2t-3)^2 (2t^2 -24t +3) =0$

 

Phần còn lại nhờ các bạn trẻ hoàn thiện nốt hen :)




#729976 Đề thi chọn đội tuyển Thanh Hóa 2021-2022

Đã gửi bởi supermember on 29-08-2021 - 11:01 trong Thi HSG cấp Tỉnh, Thành phố. Olympic 30-4. Đề thi và kiểm tra đội tuyển các cấp.

Làm thử bài 1 hen.

 

Câu a: ta sẽ chứng minh rằng với mọi số nguyên dương $n$ thì $a_n$ là số nguyên dương và $a_n   \equiv 1  \ \ ( \mod 3)$

 

Rõ ràng khẳng định đúng với $ n =1$, giả sử khẳng định đúng đến $n$, tức là $ a_n = 3k+1$ với $k$ là số nguyên không âm, khi đó :

 

$ 3a_{n+1} = (3k+2)^3 -5 = 27k^3 + 54k^2 + 36k + 3$ suy ra $ a_{n+1} = 9k^3 + 18k^2 +12k+1$ , từ đây dễ thấy $ a_{n+1} $ là số nguyên dương và còn có $ a_{n+1}   \equiv 1  \ \ ( \mod 3)$ . Khẳng định được chứng minh hoàn toàn theo nguyên lý quy nạp toán học.

 

Câu b: Từ gợi ý ban đầu từ câu a, ta thử cách đặt dãy phụ: $ a_n = 3b_n+1$ với $b_n$ là dãy số nguyên không âm

 

Rõ ràng: $b_1 =1$, và $ 9b_{n+1} +3 = (3b_n +2)^3 -5$ với mọi $n \in \mathbb{N}^{*}$ 

 

$ \implies \ \ 9b_{n+1} +3 = 27b^3_n + 54b^2_n + 36b_n +3 $

 

$ \implies \ \ b_{n+1} = 3b^3_n + 6b^2_n + 4b_n = b_n (3b^2_n + 3b_n +1) + (3b^2_n +3b_n+1) -1 $

 

$ \implies \ \ b_{n+1} +1 = (b_n +1)(3b^2_n + 3b_n +1)$

 

$ \implies \frac{1}{3b^2_n + 3b_n +1} = \frac{b_n +1}{b_{n+1} +1}$

 

Trong đó, ta dễ dàng kiểm tra $ \frac{a_k -1}{ a^2_k +a_k +1} = \frac{b_k}{3b^2_k+ 3b_k +1}$

 

Suy ra: $ \frac{a_k -1}{ a^2_k +a_k +1} = \frac{b_k (b_k +1)}{b_{k+1} +1}$

 

Tới đây suy nghĩ tiếp hen, chưa hoàn tất.




#730077 Tính $P(0)$

Đã gửi bởi supermember on 02-09-2021 - 21:34 trong Đa thức

Giỏi quá bạn già! Vậy là bài toán unsolved  hơn 2 năm đã được giải quyết!




#730119 GIẢI ĐẤU VÒNG TRÒN $2N+1$ ĐỘI

Đã gửi bởi supermember on 04-09-2021 - 12:27 trong Tổ hợp và rời rạc

Bài toán: Một cuộc đấu vòng tròn gồm $2n+1$ đội trong đó $2$ đội bất kỳ gặp nhau đúng $1$ lần. $3$ đội $X; Y; Z$ được gọi là 1 bộ ba "khó đoán" nếu $ X$ thắng $Y$, $Y$ thắng $Z$ và $Z$ thắng $X$. Biết rằng giải dấu không có trận hòa nào. Hãy tính:

 

a. Số bộ ba "khó đoán" ít nhất có thể có

b. Số bộ ba "khó đoán " nhiều nhất có thể có




#730172 GIẢI ĐẤU VÒNG TRÒN $2N+1$ ĐỘI

Đã gửi bởi supermember on 05-09-2021 - 23:37 trong Tổ hợp và rời rạc

Giải ra câu a là cũng giỏi rồi.

 

Số bộ ba khó đoán nhiều nhất có thể có là $ \frac{n(n+1)(2n+1)}{6}$, khúc xây dựng hơi khó :)

Và trong cách xây dựng thì mỗi đội cũng sẽ có số ván thắng và thua bằng nhau và bằng $n$.




#730298 TÍNH SỐ TẬP CON KHÁC RỖNG CÓ TÍCH CÁC PHẦN TỬ CHIA HẾT CHO 30

Đã gửi bởi supermember on 11-09-2021 - 12:26 trong Tổ hợp và rời rạc

Bài Toán: Cho tập $ H = \{1;2;3;....; 10 \}$. Tính số tập con khác rỗng của $H$ thỏa mãn điều kiện tích các phần tử của tập đó chia hết cho $30$.




#730308 TÍNH SỐ TẬP CON KHÁC RỖNG CÓ TÍCH CÁC PHẦN TỬ CHIA HẾT CHO 30

Đã gửi bởi supermember on 11-09-2021 - 18:07 trong Tổ hợp và rời rạc

Bạn Poset giải đúng, nhưng trình bày quá vắn tắt.
Nên viết ra chi tiết đầy đủ để các bạn khác dễ nắm bắt.




#730358 MỖI NGƯỜI ĐỀU QUEN ĐÚNG 6 NGƯỜI VÀ 2 NGƯỜI BẤT KỲ CÓ ĐÚNG 2 BẠN CHUNG

Đã gửi bởi supermember on 13-09-2021 - 17:41 trong Tổ hợp và rời rạc

Bài Toán: Tồn tại chăng một nhóm người thỏa mãn điều kiện: mỗi người đều quen biết đúng $6$ người & $2$ người bất kỳ đều có đúng $2$ bạn chung?




#730434 $A = 23p + 3^p - 4$

Đã gửi bởi supermember on 15-09-2021 - 21:46 trong Số học

Câu a:

 

Giả sử $ A$ là số chính phương, thì theo định lý Fermat nhỏ: $ 3^p \equiv 3 \pmod p$

 

$\Rightarrow A \equiv 3 -4 \pmod p \Rightarrow A \equiv -1 \pmod p$

 

Suy ra $-1$ là số chính phương $\pmod p$ , Suy ra $ (-1)^{\frac{p-1}{2}}  \equiv 1 \pmod p$

 

Tức là $p$ phải có dạng $ p = 4k+1$ với $k$ là số nguyên dương.

 

Xét số dư của $A$ khi chia cho $4$ ta có:

 

$ A = 23(4k+1) + 3^{4k+1} - 4 \equiv  23 + 3 \cdot 3^{4k} \equiv  23 + 3 \cdot 81^{k}  \equiv 23 + 3 \cdot 1   \equiv  26  \pmod 4$

 

Suy ra $ A  \equiv 2  \pmod 4$

 

Vô lý vì một số chính phương thì khi chia cho $4$ chỉ có thể dư $1$ hoặc chia hết cho $4$

 

Nên giả sử ban đầu là sai và ta có điều phải chứng minh




#730458 $A = 23p + 3^p - 4$

Đã gửi bởi supermember on 16-09-2021 - 17:15 trong Số học

Câu b: Hi vọng là giải đúng

 

Ta dễ dàng kiểm tra với $p=3$ thì $ A = 92$ không phải tích của $2$ số nguyên dương liên tiếp.

Nên ta chỉ cần xét trường hợp $p> 3$ , thì hiển nhiên : $ 3 \not| p$

 

Giả sử $A$ có thể viết dưới dạng tích của $2$ số nguyên dương liên tiếp, tức là $A$ có dạng: $ A = n(n+1)$ với $n \in \mathbb{N}^{*}$

 

$ \Rightarrow 4A +1 = (2n+1)^2$, mà theo chứng minh ở trên thì ta đã có: $ A \equiv -1 \pmod p \Rightarrow 4A +1 \equiv -3 \pmod p  $

 

$ \Rightarrow  -3 \equiv (2n+1)^2 \pmod p  $

 

Suy ra $-3$ là số chính phương $\pmod p$

 

Mà theo luật tương hỗ Gauss: Do $-3$ là số chính phương $\pmod p$ nên chỉ có thể xảy ra $2$ trường hợp:

 

Trường hợp 1: $-1$ là số chính phương $\pmod p$  và $3$ là số chính phương $\pmod p$

 

Theo tiêu chuẩn này thì $p$ phải có dạng $4k+1$ , đồng thời cũng phải có dạng $12t  \pm 1$

 

Xét trên module 4 thì suy ra $p$ phải có dạng $p = 12t+1$

 

Bây giờ do $4A+1$ là số chính phương, ta xét số dư của $4A+1$ khi chia cho $3$ , chú ý là  hiển nhiên $3 | 3^p$ :

 

$ 4A+1 \equiv A+1 \equiv 23p -4 +1  \equiv 23p \equiv 23(12t+1)  \equiv  23 \equiv 2 \pmod 3$

 

Suy ra: $ 4A+1 \equiv 2 \pmod 3$

 

Vô lý vì một số chính phương thì khi chia cho $3$ chỉ có thể dư $1$ hoặc chia hết cho $3$

Trường hợp 2: 
$-1$ không là số chính phương $\pmod p$  và $3$ không là số chính phương $\pmod p$

 

Tức là: $p$ phải có dạng $ p = 4k+3$ và $p $ không có dạng $ 12t \pm 1$, $ p$ không chia hết cho $3$

 

Thì ta dễ thấy chỉ có $1$ trường hợp là: $ p = 12t +7$

 

Cũng bằng cách xét số dư của $4A +1$ khi chia cho $3$ như ở trên , ta thấy cũng không thể xảy ra trường hợp $2$ này được.

 

Do đó giả sử ban đầu là sai và ta có điều phải chứng minh.




#730466 MỖI NGƯỜI ĐỀU QUEN ĐÚNG 6 NGƯỜI VÀ 2 NGƯỜI BẤT KỲ CÓ ĐÚNG 2 BẠN CHUNG

Đã gửi bởi supermember on 16-09-2021 - 21:31 trong Tổ hợp và rời rạc

Bài này giải bằng đếm 2 cách:

 

Nếu ta minh họa nhóm $n$ người này là những đỉnh trên một đồ thị. Sau đó, nếu $2$ người quen nhau thì ta sẽ nối $2$ đỉnh đó bởi $1$  cạnh vô hướng.

 

Cứ $2$ người mà có $1$  người quen chung thì trên đồ thị, $3$ đỉnh tương ứng với $3$ người này cùng với $2$ cạnh thể hiện sự quen biết sẽ tạo thành $1$ hình chữ $V$

 

Trong đó có $1$ đỉnh ở vị trí là đỉnh của chữ $V$ . Đỉnh này chính là minh họa tương ứng cho người quen chung đó.

 

Ta sẽ đếm $2$ cách về số chữ $V$ có thể có:

 

Cách 1: Do cứ $1$ người lại quen biết $6$ người, nên nếu chọn ra các chữ $V$ có đỉnh là 1 đỉnh cho trước thì có tổng cộng: $ \binom{6}{2} = 15$ chữ $V$ như thế.

 

Graph này có $n$ đỉnh nên tổng số chữ $V$ có thể có là : $15n$ theo quy tắc nhân.

 

Cách 2: Do cứ $2$ người bất kỳ lại có $2$ người quen chung, tức là từ $1$ cặp $2 $ đỉnh tùy ý trong graph này thì xây dựng ra được $2$ hình chữ $V$  và có $\binom{n}{2}$ cách chọn ra $1$ cặp  $2$ đỉnh tùy ý trong graph , nên theo quy tắc nhân thì  tổng số hình chữ $V$ có thể có là $2 \cdot \binom{n}{2} =n(n-1)$ .

 

Từ đây suy ra: $15n = n(n-1) \Rightarrow n =16$

 

Ta xây dựng graph như sau:

 

Phân hoạch tập $16$ đỉnh này thành $4$ tập con : $ A= \{A_1 ; A_2 ; A_3; A_4 \} ; B= \{B_1 ; B_2 ; B_3; B_4 \} ; C= \{C_1 ; C_2 ; C_3; C_4 \} ; D= \{D_1 ; D_2 ; D_3; D_4 \}$

 

Ta tiến hành nối các đỉnh này theo quy tắc sau:

 

Quy tắc 1: Các đỉnh nằm trong cùng $1$ tập hợp đều được nối với nhau:

 

Quy tắc 2: Các đỉnh khác tập hợp thì sẽ được nối với nhau nếu chúng có cùng chỉ số. Tức là các đỉnh nằm trong  $1$ bộ $4$ đỉnh $(A_k; B_k; C_k; D_k)$ sẽ được nối với nhau với mọi $k$ chạy từ $1$ đến $4$.

 

Dễ kiểm tra là với cấu hình này thì mỗi người sẽ đều có đúng $6$ người quen và $2$ người tùy ý sẽ đều có đúng $2$ người quen chung.

 

Bài toán theo đó có câu trả lời là tồn tại.




#730519 $\left\{\begin{matrix}x+y+z=3\\...

Đã gửi bởi supermember on 19-09-2021 - 06:50 trong Phương trình - Hệ phương trình - Bất phương trình

Hi vọng giải đúng. :D

 

Mấu chốt bài này là phép đặt ẩn phụ: $ a = 1-x; \ b = 1-y; \  c =1-z$

 

Với cách đặt ẩn phụ này thì hệ đã cho tương đương hệ sau:

 

$ \begin{cases} (1-a)+ (1-b)+(1-c) =3 \\ (1-a) +2(1-b)^2+ 3(1-c)^3 = 6 \\  (1-a)(1-b)+(1-b)(1-c)+ (1-c)(1-a) = (1-a)(1-b)(1-c) +2 \end{cases}$

 

$ \Leftrightarrow \begin{cases} 3 - (a+b+c) =3 \\  6 -a -4b+2b^2 -9c+9c^2- 3c^3 = 6 \\   3- 2(a+b+c) + (ab+bc+ca)  =  1 -(a+b+c) +(ab+bc+ca) -abc +2 \end{cases}$

 

$ \Leftrightarrow \begin{cases} a+b+c = 0 \\  3c^3 -9c^2 +9c -2b^2 +4b+ a = 0 \\   a+b+c =  abc \end{cases}$

 

$ \Leftrightarrow \begin{cases} a+b+c = 0 \\  3c^3 -9c^2 +9c -2b^2 +4b+ a = 0 & & (*) \\   abc = 0 \end{cases}$

 

Tới đây thì dễ rồi, do tích $abc=0$ ta chỉ cần xét $3$ trường hợp:

 

Trường hợp $1$: $ a= 0$

 

Thì khi đó hệ $(*)$ tương đương với:

 $ \begin{cases} a=0 \\  3c^3 -9c^2 +9c -2b^2 +4b= 0 \\   b+c = 0 \end{cases} \Leftrightarrow \begin{cases} a=0 \\  3c^3 -9c^2 +9c -2c^2 -4c= 0 \\   b = -c \end{cases}  \Leftrightarrow \begin{cases} a=0 \\  b = -c \\   3c^3 -11c^2 +5c = 0  \end{cases}$

 

$  \Leftrightarrow \begin{cases} a=0 \\  b = -c \\   c(3c^2 -11c +5) = 0  \end{cases}$

 

Phương trình $ 3c^2-11c+5=0 $ có 2 nghiệm: $c = \frac{11\pm \sqrt{61}}{6} $ nên suy ra trường hợp này ta thu được 3 bộ nghiệm $ \left( a; \ b; \ c \right)$:

 

$ \left( 0; \ 0; \ 0 \right); \left( 0; \  \frac{- 11-   \sqrt{61}}{6}; \  \frac{11+  \sqrt{61}}{6} \right) ; \left( 0; \  \frac{- 11+   \sqrt{61}}{6}; \  \frac{11-  \sqrt{61}}{6} \right)$

 

Trường hợp $2$: $ b= 0$

 

Thì khi đó hệ $(*)$ tương đương với:

 $ \begin{cases} b=0 \\  3c^3 -9c^2 +9c  +a= 0 \\   a+c = 0 \end{cases} \Leftrightarrow \begin{cases} b=0 \\  3c^3 -9c^2 +9c  -c= 0 \\   a = -c \end{cases}  \Leftrightarrow \begin{cases} b=0 \\  a = -c \\   3c^3 -9c^2 +8c = 0  \end{cases}$

 

$ \Leftrightarrow \begin{cases} b=0 \\  a = -c \\   c(3c^2 -9c +8) = 0  \end{cases}$

 

Do phương trình bậc $2$ :  $3c^2 -9c+8=0$ vô nhiệm nên trường hợp này chỉ có 1 bộ nghiệm  $ \left( a; \ b; \ c \right)$ là $ \left( 0; \ 0; \ 0 \right)$

 

(Lưu ý ở đây ta giải trên tập số thực, nếu chấp nhận giải trên tập số phức thì sẽ có thêm vài nghiệm, cụ thể là sẽ có thêm $2$ bộ nghiệm phức )

 

Trường hợp $3$: $ c= 0$

 

Thì khi đó hệ $(*)$ tương đương với:

 $ \begin{cases} c=0 \\  -2b^2 +4b+ a = 0 \\   a+b = 0 \end{cases} \Leftrightarrow \begin{cases} c=0 \\  -2b^2 +4b- b = 0 \\   a = -b \end{cases} \Leftrightarrow \begin{cases} c=0 \\  a = -b \\   2b^2 -3b=0  \end{cases}$

 

Do phương trình bậc $2$ :  $2b^2 -3b=0$ có $2$ nghiệm :$ b=0 ; \ b = \frac{3}{2}$ nên trường hợp này ta thu được 2 bộ nghiệm  $ \left( a; \ b; \ c \right)$ là : $ \left( 0; \ 0; \ 0 \right) $và $ \left( \frac{-3}{2}; \ \frac{3}{2} ; \ 0 \right)$

 

Từ đây suy ra hệ đã cho có $4$ bộ nghiệm $ \left( x; \ y; \ z \right)$ là:

 

$ \left( 1; \ 1; \ 1 \right) ; \left( \frac{5}{2}; \ \frac{-1}{2} ; \ 1 \right) ; \left( 1; \  \frac{17+   \sqrt{61}}{6}; \  \frac{-5 - \sqrt{61}}{6} \right); \left( 1; \  \frac{17-   \sqrt{61}}{6}; \  \frac{-5+  \sqrt{61}}{6} \right)$

 

Bài toán theo đó được giải quyết hoàn toàn.




#730548 Đề thi chọn đội tuyển tỉnh Đồng Tháp 2021

Đã gửi bởi supermember on 19-09-2021 - 20:42 trong Thi HSG cấp Tỉnh, Thành phố. Olympic 30-4. Đề thi và kiểm tra đội tuyển các cấp.

Bài 2:   Cho $a,b,c$ là các số nguyên dương, trong đó $a,b$ nguyên tố cùng nhau và $\frac{b}{c}+\frac{c}{a}$ là số nguyên.

 

Chứng minh rằng $a$ là số chính phương

 

 

 

Hi vọng là giải đúng : :icon6:

 

Ta giả sử $ \frac{b}{c}+ \frac{c}{a} = x$  với $ x \in \mathbb{N}^{*}$

 

$ \Leftrightarrow ba + c^2 = xca \Leftrightarrow a(xc-b) = c^2  $ $ (*)$

 

Do đó dễ thấy : $a; \ xc-b $ đều là những số nguyên dương, có tích là bình phương của $1$ số nguyên dương $ (**)$

 

Để chứng minh khẳng định bài toán, thực chất chỉ cần chứng minh $2$ số $ a; \ xc-b$ nguyên số cùng nhau. Khi đó, thì $a; \ xc-b$ sẽ đều là bình phương của $1$ số nguyên dương. Điều này suy ra trực tiếp từ định lý cơ bản của số học về phân tích $1$ số nguyên dương ra thành tích của các thừa số nguyên số. (Phân tích thành lũy thừa nguyên tố)

 

Nhắc lại nội dung định lý:

 

Mọi số nguyên dương đều biểu diễn được một cách duy nhất dưới dạng tích của các số nguyên tố, trong đó các thừa số nguyên tố được viết theo thứ tự không giảm.

 

Trở lại việc giải bài toán:

 

Thật vậy, giả sử $p$ là ước số nguyên tố tùy ý của $a$ thì $ p|a$

 

Từ $(*)$ ta có $ a| c^2$ nên suy ra $ p|c$ 

 

$ \Rightarrow  p|xc  \Rightarrow   p \not | xc-b$ do $a;b$ nguyên tố cùng nhau.

 

Suy ra $xc-b$ không chia hết cho mọi ước nguyên tố của $a$

 

Do đó $ xc-b; a$ phải là 2 số nguyên tố cùng nhau.

 

Suy ra: $xc-b; a$ đều là bình phương của $1$ số nguyên dương 

 

Bài toán theo đó được giải quyết hoàn toàn.




#730582 Tính số bộ $(x_1; x_2;...; x_n)$ thỏa mãn hiệu của tích và tổng chi...

Đã gửi bởi supermember on 21-09-2021 - 11:07 trong Tổ hợp và rời rạc

Bài Toán: Tính số bộ số $(x_1;x_2;...; x_n)$ thỏa mãn: $ x_k \in \{0;1;2\} $ với mọi $ k = \overline{1;n}$ và: $ \prod_{k=1}^{n} x_k - \sum_{k=1}^{n} x_k$ là số chia hết cho $3$




#730689 Đề chọn đội tuyển VMO Hà Tĩnh 2021-2022

Đã gửi bởi supermember on 25-09-2021 - 17:32 trong Thi HSG cấp Tỉnh, Thành phố. Olympic 30-4. Đề thi và kiểm tra đội tuyển các cấp.

 

Bài 2. (5 điểm) 

Cho đa thức $P(x)$ có các hệ số nguyên, giả sử các phương trình $P(x)=1, P(x)=2$ và $P(x)=3$ theo thứ tự mỗi phương trình có ít nhất một nghiệm nguyên theo lần lượt $x_1,x_2,x_3$.

a) Chứng minh rằng: $x_1,x_2,x_3$ là các nghiệm nguyên duy nhất của các phương trình trên.

b) Chứng minh rằng: phương trình $P(x)=5$ không có hơn một nghiệm nguyên. 

 

 

Hi vọng là giải đúng: :D  :D  :D 

Câu a: Ta cần dùng đến tính chất cơ bản của đa thức hệ số nguyên: với $x;y$ là những số nguyên phân biệt thì ta có: $ x-y | P(x)-P(y)$

Sử dụng tính chất này, kèm theo chú ý: hiển nhiên $ x_1 ;\  x_2 ; \  x_3$ phải là những số nguyên đôi một phân biệt, ta có:

 

$ \begin{cases} x_2 - x_1 | P(x_2)- P(x_1) \\ x_3 -  x_2 | P(x_3) - P(x_2) \end{cases}  \Rightarrow \begin{cases} x_2 - x_1 | 1 \\ x_3 -  x_2 | 1 \end{cases}  $

 

$ \Rightarrow  x_2- x_1 ; x_3 - x_2 \in \{ \pm 1 \}$

 

Trường hợp 1:  $x_2 - x_1 =1$  thì ta phải có $ x_3 - x_2 = 1$ vì nếu $ x_3 - x_2 =  -1$ thì suy ra : $ (x_2 - x_1) + (x_3 - x_2 ) = 0 \Rightarrow x_3 = x_1 $

Điều này vô lý.

 

Trường hợp 2:  $x_2 - x_1 = -1$ thì  bằng chứng minh tương tự, ta phải có: $ x_3 - x_2 =  -1$ 

 

Tức là $x_1; \  x_2; \  x_3$ tạo thành cấp số cộng công sai  $1$ hoặc $-1$.

 

Trước tiên, ta đi chứng minh rằng giá trị $x_1$ nếu có tồn tại, thì là duy nhất:

 

Thật vậy, Giả sử ngoài $x_1$ thì phương trình $ P(x)=1$ còn có thêm nghiệm nguyên $ x_1^{'}$

 

* Nếu $ x_2 - x_1 = 1$ thì $ x_2 - x_1^{'} = -1$, mà $ x_3 - x_2 =1$, suy ra $ (x_3 - x_2) +  (x_2 - x_1^{'}) = 0 \Rightarrow  x_3 = x_1^{'}$ (Vô lý)

* Nếu  $x_2 - x_1 = -1$ thì $ x_2 - x_1^{'} = 1$, mà $ x_3 - x_2 =-1$, suy ra $ (x_3 - x_2) +  (x_2 - x_1^{'}) = 0 \Rightarrow  x_3 = x_1^{'}$ (Vô lý)

 

Nên giá trị $x_1$ nếu tồn tại theo giải thiết bài toán, thì là duy nhất.

 

Để chứng minh khẳng định bài toán, ta chỉ cần chứng minh: 

 

Không thể xảy ra trường hợp $2$ bộ số $ (x_1; x_1+1; x_1 +2); (x_1; x_1 - 1; x_1 -2)$ đều lần lượt là nghiệm của $3$ phương trình: $P(x)=1; \ P(x) =2 ; \ P(x) =3$

 

Nhưng may mắn là chứng minh điều này không khó vì nếu giả sử xảy ra trường hợp này thì ta có:

 

$ ( x_1+2) - (x_1 -1) | P( x_1+2) - P(x_1 -1) \Rightarrow  3| 3-2 \Rightarrow  3| 1$ (Vô lý)

 

Do đó, khẳng định bài toán được chứng minh hoàn toàn. Tức là $3$ phương trình: $P(x)=1; \ P(x) =2 ; \ P(x) =3$ chỉ có $1$ bộ nghiệm duy nhất $(x_1; x_2;x_3)$ có dạng $ (x_1; x_1+1; x_1 +2)$ hoặc $ (x_1; x_1 - 1; x_1 -2)$

 

Câu b: Ý tưởng cũng là hoàn toàn tương tự, ta giả sử $x_4$ là nghiệm nguyên của phương trình $ P(x)=5$

 

ta có $ \begin{cases} x_4 - x_3 |  2  \\ x_4 -  x_2 | 3 & (\bigstar)  \end{cases} $

 

Dễ thấy không thể xảy ra trường hợp: $ x_4 - x_3 = \pm 1$

 

Thật vậy, nếu $ x_4 -x_3 = -1 $ thì dễ thấy là không thể xảy ra trường hợp $ x_3 - x_2 =1$ vì nếu như vậy thì: $ (x_4 -x_3) +  (x_3 - x_2) =0 \Rightarrow x_4 = x_2$ (Vô lý). Còn nếu $  x_3 - x_2 = -1$ suy ra $x_4 - x_2 = -2$, nên theo $ (\bigstar) $ thì $ -2 |3$ (vô lý)

 

Ta chứng minh hoàn toàn tương tự để có: không thể xảy ra trường hợp: $ x_4 -x_3 =  1 $

 

Do đó, chỉ có thể xảy ra trường hợp: $ x_4 - x_3 = \pm 2$

 

Đến đây thì gần xong rồi, ta giả sử ngoài nghiệm nguyên $x_4$ thì phương trình $P(x) =5$ còn có thêm nghiệm nguyên $ x_4^{'}$

 

* Nếu $ x_4 - x_3 = 2$ thì suy ra $ x_4^{'} - x_3 = -2$

 

Trường hợp $ x_3 - x_1 = 2 $ thì : $ ( x_4^{'} - x_3 ) + (x_3 - x_1) = 0 \Rightarrow x_4^{'} = x_1$ (Vô lý)

Trường hợp $  x_3 - x_1 = -2 $ thì : $ ( x_4 - x_3 ) + (x_3 - x_1) = 0 \Rightarrow x_4= x_1$ (Vô lý)

 

Nên không thể xảy ra trường hợp này .

 

* Nếu $ x_4 - x_3 = - 2$ thì suy ra $ x_4^{'} - x_3 = 2$

 

Chứng minh tương tự, ta cũng chỉ ra được không thể xảy ra trường hợp này.

 

Mâu thuẫn này chứng tỏ điều giả sử ban đầu về sự tồn tại của $x_4^{'}$ là sai, và giá trị $x_4$ ( nếu có tồn tại)  thì sẽ là duy nhất.

Khẳng định bài toán theo đó được chứng minh hoàn toàn.




#730729 Đề thi chọn đội tuyển Học sinh giỏi môn Toán tỉnh Thanh Hóa năm 2017 - 2018.

Đã gửi bởi supermember on 27-09-2021 - 18:02 trong Thi HSG cấp Tỉnh, Thành phố. Olympic 30-4. Đề thi và kiểm tra đội tuyển các cấp.

Bài 5. Chứng minh rằng: $\sum_{k=0}^{n}2^{k}C_{n}^{k}C_{n-k}^{\left [ \frac{n-k}{2} \right ]}=C_{2n+1}^{n}.$

*Đề thi có tham khảo ở link sau: http://olympictoanho...-2017-2018.html

 

Hi vọng là giải đúng:  :D  :D  :D

 

Ta để ý : $ C_{2n+1}^{n} = C_{2n}^{n} + C_{2n}^{n-1}$ , chính là  tổng của hệ số của $x^{0}$ và $ \frac{1}{x^2}$ trong khai triển $ \left( x + \frac{1}{x} \right)^{2n}$  

 

Nên một cách tự nhiên, ta đi chứng minh vế trái cũng bằng tổng của hệ số của $x^{0}$ và $ \frac{1}{x^2}$ trong khai triển $ \left( x + \frac{1}{x} \right)^{2n}$  

 

Ở đây do có dấu phần nguyên nên ta sẽ phá dấu phần nguyên này bằng cách lần lượt xét tính chẵn lẻ của $n;k$

 

Ta xét trường hợp : $n$ chẳn, $n = 2m $ với $ m \in \mathbb{N}^{*}$, trường hợp $n $ lẻ ta làm hoàn toàn tương tự.

 

Khi $n = 2m$ thì vế trái của đẳng thức đã cho bằng:

 

$ \sum_{k=0}^{2m} 2^{k} C_{2m}^{k} C_{2m-k}^{ \left[ \frac{2m-k}{2} \right] } =  \sum_{k=0}^{m} 2^{2k} C_{2m}^{2k} C_{2m-2k}^{ \left[ \frac{2m-2k}{2} \right] } +  \sum_{k=0}^{m-1} 2^{2k+1} C_{2m}^{2k+1} C_{2m-2k-1}^{ \left[ \frac{2m-2k-1}{2} \right] } $

$= \sum_{k=0}^{m} 2^{2k} C_{2m}^{2k} C_{2m-2k}^{m-k } +  \sum_{k=0}^{m-1} 2^{2k+1} C_{2m}^{2k+1} C_{2m-2k-1}^{m-k-1}  $  $ (\bigstar )$

 

(Xét tính chẳn lẻ của $k$ để tách $1$ tổng thành $2$ tổng con )

 

Mặt khác, ta xét khai triển của $ \left( x + \frac{1}{x} \right)^{2n}$  :

 

$ \left( x + \frac{1}{x} \right)^{2n}  = \left( x^2 +2 + \frac{1}{x^2} \right)^n =  \left( x + \frac{1}{x} \right)^{2n}  = \left( x^2 +2 + \frac{1}{x^2} \right)^{2m} =\sum_{k=0}^{2m} 2^{k} C_{2m}^{k} \left(x^2 + \frac{1}{x^2} \right)^{2m-k}$

 

$ = \sum_{k=0}^{m} 2^{2k} C_{2m}^{2k} \left(x^2 + \frac{1}{x^2} \right)^{2m-2k} + \sum_{k=0}^{m-1} 2^{2k+1} C_{2m}^{2k+1} \left(x^2 + \frac{1}{x^2} \right)^{2m-2k-1} $

 

Đến đây ta có $2$ nhận xét đơn giản:

 

Nhận xét 1: trong khai triển $  \left(x^2 + \frac{1}{x^2} \right)^{2m-2k}$ không thể có phần tử $ \frac{1}{x^2}$, chỉ có phần tử $ x^0$ và hệ số của phần tử này bằng: $ C_{2m-2k}^{m-k}$

 

Nhận xét 2: trong khai triển $  \left(x^2 + \frac{1}{x^2} \right)^{2m-2k-1}$ không thể có phần tử $ x^0$, chỉ có phần tử $ \frac{1}{x^2}$ và hệ số của phần tử này bằng: $ C_{2m-2k-1}^{m-k-1}$

 

Từ $2$ nhận xét này, ta suy ra: tổng hệ số của $x^{0}$ và $ \frac{1}{x^2}$ trong khai triển $ \left( x + \frac{1}{x} \right)^{2n}$  chính bằng:

 

$ \sum_{k=0}^{m} 2^{2k} C_{2m}^{2k} C_{2m-2k}^{m-k} + \sum_{k=0}^{m-1} 2^{2k+1} C_{2m}^{2k+1} C_{2m-2k-1}^{m-k-1} $ $( \bigstar \bigstar)$

 

Từ $ (\bigstar ) ; ( \bigstar \bigstar)$ ta thấy rằng vế trái $=$ vế phải và khẳng định bài toán theo đó được chứng minh hoàn toàn :)




#730953 Tính số bộ $(x_1; x_2;...; x_n)$ thỏa mãn hiệu của tích và tổng chi...

Đã gửi bởi supermember on 04-10-2021 - 21:04 trong Tổ hợp và rời rạc

Bài này supermember sẽ trình bày ý tưởng đã, thực sự thì nếu trình bày full lời giải dựa trên ý tưởng này thì rất dài nên hi vọng có người nào đó có thể đưa ra cách làm tốt hơn.

 

Trước tiên, thay vì trực tiếp đếm số bộ số cần tìm, ta sẽ lần lượt đếm những bộ số đơn giản hơn:

 

Bước 1: Ta đi tính số bộ số $ (x_1; x_2; ...; x_n)$ với $ x_k \in \{1;2 \}$ với mọi $k$ chạy từ $1$ đến $n$ sao cho : $ 3 | x_1 + x_2 + \cdots + x_n$

Giả sử trong bộ $n$ số này có $k$ số $1$ và $l$ số $2$ thì ta có:

 

$ 0 \leq k; l \leq n ; \  k + l =n ; \ 3 | k+ 2l$

Ta xét các trường hợp sau:

 

Trường hợp $1$: $ n \equiv 0 \pmod 3 $

Khi đó, dễ thấy chỉ có thể xảy ra trường hợp: $ k \equiv l \equiv 0 \pmod 3$

 

Nên từ đây dễ thấy là $k$ sẽ có thể nhận các các giá trị: $ 0; 3 ; …; n$

 

Do đó, theo quy tắc cộng, thì số cách chọn ra bộ số trong trường hợp này là:

 

$ \binom{n}{0} + \binom{n}{3} + \cdots + \binom{n}{n}$

 

Trường hợp $2$: $ n \equiv 1 \pmod 3 $

 

Khi đó, dễ thấy chỉ có thể xảy ra trường hợp: $ k \equiv l \equiv 2 \pmod 3$

 

số cách chọn ra bộ số trong trường hợp này là:

 

$ \binom{n}{2} + \binom{n}{5} + \cdots + \binom{n}{n -2}$

 

Trường hợp $3$: $ n \equiv 2 \pmod 3 $

 

Khi đó, dễ thấy chỉ có thể xảy ra trường hợp: $ k \equiv l \equiv 1 \pmod 3$

 

số cách chọn ra bộ số trong trường hợp này là:

 

$ \binom{n}{1} + \binom{n}{4} + \cdots + \binom{n}{n -1}$

 

Mặt khác, theo chuyên đề đẳng thức tổ hợp của diễn đàn, thì ta có $3$ đẳng thức sau:

 

$ \binom{n}{0} + \binom{n}{3} + \binom{n}{6} + \binom{n}{9} + \cdots = \frac{1}{3} \cdot \left( 2^n + 2 \cos \frac{ \pi n}{3} \right)$   

$ \binom{n}{1} + \binom{n}{4} + \binom{n}{7} + \binom{n}{10} + \cdots = \frac{1}{3} \cdot \left( 2^n + 2 \cos \frac{n-2}{3} \pi \right)$   

$ \binom{n}{2} + \binom{n}{5} + \binom{n}{8} + \binom{n}{11} + \cdots = \frac{1}{3} \cdot \left( 2^n + 2 \cos \frac{n-4}{3} \pi \right)$   

 

Bước 2: Ta để ý : chỉ cần $1$ trong số $n$ số $ x_1; x_2 ;...; x_n$ bằng $0$ thì tích của $n$ số này bằng $0$, chia hết cho $3$

 

Và việc thêm hay bớt $1$ số $0$ trong tổng $ x_1+ x_2 +...+ x_n$ không ảnh hưởng đến số dư của tổng này trong phép chia cho $3$

 

Chẳng hạn như từ $1$ bộ $k$ số $ (x_1; x_2; ...; x_k)$  mà trong đó mỗi số $ x_1 ; x_2;... ; x_k \in \{1;2 \}$ với $k < n$ thì ta có thể cho tương ứng với: $\binom{n}{n-k} = \binom{n}{k}$ bộ số  $ (x_1; x_2; ...; x_n)$ thỏa yêu cầu bài toán mà trong đó mỗi số $ x_1 ; x_2;... ; x_n \in \{ 0; 1;2 \}$ . Hiểu đơn giản là có $ \binom{n}{n-k} $cách chọn vị trí cho $n-k$ số $0$ để điền vào $n$ ô trống cho trước.  Do đó, bằng cách thiết lập quan hệ truy hồi, ta có thể tính ra:

 

Số bộ số $ (x_1; x_2; ...; x_n)$  thỏa yêu cầu bài toán mà trong $n$ số này có ít nhất $1$ số $0$. Ý tưởng đơn giản là dùng quy tắc cộng và xét lần lượt: trường hợp có đúng $1$ số $0$; trường hợp có đúng $2$ số $0$; ...; trường hợp có đúng $n$ số $0$ . Hiển nhiên sẽ không thể xảy ra trường hợp tổng $ x_1+ x_2 +...+ x_n$ chia hết cho $3$ và không có số $0$ nào trong tổng này vì khi đó thì dễ kiểm tra tích $ x_1 x_2  \cdots x_n$ không chia hết cho $3$, vô lý vì không đáp ứng giả thiết ban đầu của bài toán là hiệu của tổng và tích $n$ số này chia hết cho $3$.

 

Bước 3: Ta đi tính số bộ số $ (x_1; x_2; ...; x_n)$ với mỗi số $ x_1; x_2 ; ...; x_n \in \{1;2 \}$ và tổng $ x_1 + x_2 +...+ x_n$ này không chia hết cho $3$

Không còn cách nào khác là cũng đi vét cạn theo môđulô $3$ mà thôi.

 

Trường hợp $1$ $ n \equiv 0 \pmod 3 $

Ta thử xét hệ phương trình đồng dư:

 

$ \begin{cases} n \equiv 0 \pmod 3 \\ k+2l \equiv 1 \pmod 3 \\ 2^l \equiv 1 \pmod 3 \end{cases}$ hoặc $ \begin{cases} n \equiv 0 \pmod 3 \\ k+2l \equiv 2 \pmod 3 \\ 2^l \equiv 2 \pmod 3 \end{cases}$

 

tức là: $ \begin{cases} n=k+l \equiv 0 \pmod 3 \\ k+2l \equiv 1 \pmod 3 \\ l \equiv 0 \pmod 2 \end{cases}$ hoặc $ \begin{cases} n=k+l \equiv 0 \pmod 3 \\ k+2l \equiv 2 \pmod 3 \\ l \equiv 1 \pmod 2 \end{cases}$

 

$ \Leftrightarrow \begin{cases} n=k+l \equiv 0 \pmod 3 \\ k \equiv 2 \pmod 3 \\ l \equiv 1 \pmod 3 \\ l \equiv 0 \pmod 2 \end{cases}$ hoặc $\begin{cases} n=k+l \equiv 0 \pmod 3 \\ k \equiv 1 \pmod 3 \\ l \equiv 2 \pmod 3 \\ l \equiv 1 \pmod 2 \end{cases}$

 

$ \Leftrightarrow \begin{cases} n=k+l \equiv 0 \pmod 3 \\ k \equiv 2 \pmod 3 \\ l \equiv 4 \pmod 6 \end{cases}$ hoặc $\begin{cases} n=k+l \equiv 0 \pmod 3 \\ k \equiv 1 \pmod 3 \\ l \equiv 5 \pmod 6 \end{cases}$

 

$ \Leftrightarrow \begin{cases} n=k+l \equiv 0 \pmod 6 \\ k \equiv 2 \pmod 6 \\ l \equiv 4 \pmod 6 \end{cases}$ hoặc $\begin{cases} n=k+l \equiv 3 \pmod 6 \\ k \equiv 5 \pmod 6 \\ l \equiv 4 \pmod 6 \end{cases}$ hoặc $\begin{cases} n=k+l \equiv 0 \pmod 6 \\ k \equiv 1 \pmod 6 \\ l \equiv 5 \pmod 6 \end{cases}$ hoặc $\begin{cases} n=k+l \equiv 3 \pmod 6 \\ k \equiv 4 \pmod 6 \\ l \equiv 5 \pmod 6 \end{cases}$

 

$ \Leftrightarrow \begin{cases} n=k+l \equiv 0 \pmod 3 \\ l \equiv 4;5 \pmod 6 \end{cases}$ 

 

Tức là với $ n \equiv 0 \pmod 3$  thì số bộ số cần tính là tổng của $2$ tổng con:

 

$ \binom{n}{4} + \binom{n}{10} + \binom{n}{16} + \binom{n}{22} + \cdots $

 

Và $ \binom{n}{5} + \binom{n}{11} + \binom{n}{17} + \binom{n}{23} + \cdots $

 

Nhưng những tổng con này đều có thể tính ra chi tiết được theo $n$ vì theo kết quả đã nêu ra trong chuyên đề đẳng thức tổ hợp thì ta có kết quả tổng quát:

 

$ \sum_{ r \geq 0}^{} \binom{n}{j+rk} = \frac{2^n}{k} \sum_{m=0}^{k-1} \left( \cos \frac{ m \pi}{k} \right)^{n}  \cos \frac{ (n-2j)m \pi}{k}  (\bigstar)$

 

Ta làm tương tự để tính với các trường hợp: $ n \equiv 1 \pmod 3$ ;  $ n \equiv 2 \pmod 3$

 

Bước 4: Số bộ số thỏa yêu cầu bài toán là tổng các bộ số thuộc $2$ nhóm:

 

- Nhóm $1$: Nhóm các bộ số thỏa mãn điều kiện bài toán và $ 3 | x_1 + x_2 + \cdots + x_n$ (đã tính ra chi tiết từ bước $2$)

 

Nhóm $2$: Nhóm các bộ số thỏa mãn điều kiện bài toán và $ 3 \not | x_1 + x_2 + \cdots + x_n$ (đã tính ra chi tiết từ bước $3$)

 

Theo đó thì bài toán được giải quyết hoàn toàn.

 

Lời nhờ vả: Trong chuyên đề đẳng thức tổ hợp, kết quả $(\bigstar)$ được nêu ra như $1$ bài tập tự giải, các bạn hãy giải bài toán này chi tiết. Đồng thời hoàn thiện lời giải trên. Việc còn lại là các công đoạn thu gọn tổng cần tính để ra được kết quả cuối cùng theo $n$ . :D  :D  :D Sau vài ngày nghỉ dịch thì nay supermember quay trở lại nhịp sống bình thường. Xin nhường những việc này cho các bạn :)




#731136 Đề thi chọn đội tuyển Học sinh giỏi môn Toán tỉnh Bình Thuận năm 2017 - 2018

Đã gửi bởi supermember on 13-10-2021 - 22:02 trong Thi HSG cấp Tỉnh, Thành phố. Olympic 30-4. Đề thi và kiểm tra đội tuyển các cấp.

 

Bài 1. Tìm tất cả các cặp số nguyên $(x,y)$ thỏa mãn:

$$x^4+x^3+x^2+2x=y^2+y$$

 

 

 

 

Hi vọng là giải đúng :)

 

Ta viết phương trình đã cho dưới dạng sau:

 

$ 4x^4 + 4x^3 + 4x^2 + 8x = 4y^2 +4y$

 

$ \Leftrightarrow  4x^4 + 4x^3 + 4x^2 + 8x  +1 = 4y^2 +4y +1$

$ \Leftrightarrow  4x^4 + 4x^3 + 4x^2 + 8x  +1 = (2y+1)^2$

 

Suy ra: $4x^4 + 4x^3 + 4x^2 + 8x  +1 $ là bình phương của một số tự nhiên lẻ.

 

Ta xét trường hợp $ x \geq 7$ thì khi đó dễ thấy:

 

$  (2x^2 +x)^2 <  4x^4 + 4x^3 + 4x^2 + 8x  +1 < (2x^2 +x+1)^2$ tức là khi $ x \ge 7$  thì $ 4x^4 + 4x^3 + 4x^2 + 8x  +1 $ không thể là bình phương của một số tự nhiên.

 

Hoàn toàn tương tự, với $ x \leq -3$ thì:   $  (2x^2 +x)^2 <  4x^4 + 4x^3 + 4x^2 + 8x  +1 < (2x^2 +x+1)^2$ và $ 4x^4 + 4x^3 + 4x^2 + 8x  +1 $ không thể là bình phương của một số tự nhiên.

 

Như vậy ta chỉ cần thử trực tiếp các giá trị của $ 4x^4 + 4x^3 + 4x^2 + 8x  +1$ với $x$ là số nguyên chạy từ $-2$ đến $6$

 

Dễ thấy với $ x = 0$ ; $ x =6$ thì $ 4x^4 + 4x^3 + 4x^2 + 8x  +1$ nhận giá trị là bình phương của số tự nhiên lẻ.  

 

Cụ thể: với $x =0$ thì  $ 4x^4 + 4x^3 + 4x^2 + 8x  +1 = 1 = 1^2$

 

Với $x =6$ thì  $ 4x^4 + 4x^3 + 4x^2 + 8x  +1 = 6241 = 79^2$

 

Nên từ đây ta dễ dàng giải ra được $4$ cặp nghiệm nguyên $(x;y)$ của phương trình đã cho là: $ (0;0); \ (0;-1); \ (6;39); \ (6; -40)$

 

Bài toán theo đó được giải quyết hoàn toàn. 




#731141 ĐỀ THI HSG 12 ĐỒNG NAI HỆ CHUYÊN

Đã gửi bởi supermember on 14-10-2021 - 12:19 trong Thi HSG cấp Tỉnh, Thành phố. Olympic 30-4. Đề thi và kiểm tra đội tuyển các cấp.

 

 

Bài 5. Cho $m,n$ là các số tự nhiên thoả $4m^3+m=12n^3+n.$ Chứng minh $m-n$ là lập phương của một số nguyên

 

Hi vọng là giải đúng.  :icon6:  :icon6:  :icon6:

 

Từ giả thiết bài toán, ta dễ thấy $ m \geq n \geq 0$

 

Trường hợp $1$: $n=0$

Khi đó dễ thấy $ m =0 \implies m-n =0 = 0^3$ là lập phương của $1$ số tự nhiên.

 

Trường hợp $2$: $ n \geq 1$

Khi đó dễ thấy $ m >n \implies m-n \geq 1$

 

Ta viết lại đẳng thức đã cho dưới dạng như sau: $ 4(m^3 - n^3 ) + (m-n) = 8n^3$

 

$ \Leftrightarrow 4(m-n)( m^2 +n^2+mn) + (m-n) = (2n)^3$

 

$ \Leftrightarrow (m-n)( 4m^2 +4n^2+4mn +1) = (2n)^3$

 

$ \Leftrightarrow (m-n) \cdot \left( 4(m-n)^2 + 12(m-n)n+12n^2 +1 \right) = (2n)^3$  $( \bigstar )$

 

Từ đây ta dễ thấy là để chứng minh khẳng định bài toán, ta chỉ cần đi chứng minh:

 

$ \gcd \left(  m-n; \  4(m-n)^2 + 12(m-n)n+12n^2 +1 \right) =1$ $( \bigstar \bigstar)$

 

Do $(2n)^3$ khi viết dưới dạng phân tích thành tích các thừa số nguyên tố thì sẽ có dạng: $ (2n)^3 = p^{3 \alpha_1}_1 \cdot p^{3 \alpha_2}_2 \cdot p^{3 \alpha_3}_3 \cdots p^{3 \alpha_k}_k$ trong đó $p_1 ; \ p_2; \cdots ; p_k$ là những số nguyên tố phân biệt và $ \alpha_1; \ \alpha_2; \ \cdots; \alpha_k$ là những số nguyên dương. Nên nếu chứng minh được $( \bigstar \bigstar)$ thì rõ ràng $m-n$ phải có dạng : $ p^{3 \alpha_{i_1}}_{i_1} \cdot p^{3 \alpha_{i_2}}_{i_2} \cdot p^{3 \alpha_{i_3}}_{i_3}\cdots p^{3 \alpha_{i_m}}_{i_m}$ trong đó tập hợp các chỉ số $ \{ i_1; \ i_2 ; \ i_3; \cdots ; \ i_m \}$ này là tập con của tập hợp các chỉ số $\{ 1; \ 2; \ 3; \cdots ; \ k \}$. (Lập luận này vẫn đúng trong trường hợp $m-n =1$ ; tương ứng với trường hợp $ \{ i_1; \ i_2 ; \ i_3; \cdots ; \ i_m \} = \emptyset $)

 

Nhưng chứng minh điều này thật ra không khó:

 

Ta giả sử tồn tại số nguyên tố $p$ là ước chung của $ m-n; \  4(m-n)^2 + 12(m-n)n+12n^2 +1$

 

Do $p | m-n$ nên từ $( \bigstar )$ suy ra $ p | (2n)^3 \implies p|2n  \implies p \not | 3 \cdot (2n)^2 +1 \implies p \not | 12n^2 +1 \implies  p \not | 4(m-n)^2 + 12(m-n)n+12n^2 +1$

Điều vô lý này chứng tỏ giả sử ban đầu về sự tồn tại của $p$ là sai.

 

Ta được:  $ \gcd \left(  m-n; \  4(m-n)^2 + 12(m-n)n+12n^2 +1 \right) =1$ , kết hợp với $( \bigstar )$, suy ra $m-n$ phải là lập phương của một số tự nhiên.

 

Khẳng định bài toán theo đó được chứng minh hoàn toàn. :icon6:




#731274 LÂM ĐỒNG 2022

Đã gửi bởi supermember on 23-10-2021 - 14:05 trong Thi HSG cấp Tỉnh, Thành phố. Olympic 30-4. Đề thi và kiểm tra đội tuyển các cấp.

 

 
Câu 3. (3.0 điểm) Cho $a, b, c$ là các số thực dương thỏa mãn $2abc=2a+4b+7c$
Tìm giá trị nhỏ nhất của biểu thức $P =a+b+c$.
 
 

 

Hi vọng là giải đúng  :closedeyes:

 

Từ điều kiện bài toán, ta suy ra: $ 2a(bc-1) = 4b+7c$. Nên dễ thấy là $bc >1$ và $ a= \frac{4b+7c}{2(bc-1)}$.

 

Suy ra : $ a+b +c =  \frac{4b+7c}{2(bc-1)} + b + c$  $(1)$

 

Do $ bc > 1$ , suy ra ta phải có: $ b > \frac{1}{c}$. Ta sẽ cố định $c$, đồng thời đi khảo sát hàm số: $ f(x) = \frac{4x+7c}{2(xc-1)} + x + c$ trên $ \left( \frac{1}{c}; \ + \infty \right)$.

 

Lấy đạo hàm $ f^{'} (x) = \frac{ 4 \cdot 2(xc-1) - (4x+7c) \cdot 2c}{ 4(xc-1)^2} +1 = 1 + \frac{ -4 - 7c^2}{2(xc-1)^2}$
 

Do đó $ f^{'} (x) = 0 \Leftrightarrow 2(xc-1)^2 = 4+ 7c^2  \Leftrightarrow \sqrt{2} (xc-1) =  \sqrt{4+7c^2}$ $( \bigstar)$

(Do trên miền giá trị $x$ đang xét thì hiển nhiên là : $ xc-1 >0$)

 

Tức là : $ f^{'} (x) = 0 \Leftrightarrow x = x_0 = \frac{1}{c} + \sqrt{ \frac{4+7c^2}{2c^2}}$

 

Từ dáng điệu của hàm số, ta dễ dàng thấy rằng $f(x)$ là hàm giảm trên $ \left( \frac{1}{c}; \  x_0 \right]$ và là hàm tăng trên $ \left[ x_0; \ + \infty \right)$.

 

Nên từ đây, ta suy ra $ f(b) \geq f(x_0 ) = \frac{4 x_0 +7c}{2(x_0 \cdot c-1)} + x_0 + c$   $(2)$

 

Bằng tính toán cụ thể, ta ra được kết quả $ f(x _0 ) = c + \frac{3}{c} + \frac{ \sqrt{8 + 14c^2}}{c} = g(c)$  $(3)$

 

Lưu ý là trong tính toán thì phải sử dụng đến đẳng thức $( \bigstar)$ thì khai triển sẽ đỡ cồng kềnh.

 

Do đó, giờ công việc của ta thực chất chỉ là đi khảo sát hàm $ g(c) = c + \frac{3}{c} + \frac{ \sqrt{8 + 14c^2}}{c}$ trên $(0 ; + \infty)$.

 

Đây là bài toán khảo sát hàm một biến, có lẽ không khó. Ta tạm dừng nghỉ ngơi. Hẹn các bạn trong giây lát.

 

Tiếp tục công việc:

 

Ta tính đạo hàm $ g^{'} (c) = 1 - \frac{3}{c^2} - \frac{8}{ c^2 \sqrt{8+14c^2}}$

 

Ta đi giải phương trình $  g^{'} (c) = 0$. Thật vậy, đặt ẩn phụ $ t = c^2 ( t>0)  $ thì dễ thấy là phương trình ta cần giải tương đương với:

 

$ t - 3 = \frac{8}{ \sqrt{8+14t}}$

 

Tức là : $ t> 3$ và $ (t-3)^2 (8+14t) =8^2$  $ ( \bigstar \bigstar)$

 

Phương trình $( \bigstar \bigstar)$ chỉ có $1$ nghiệm duy nhất lớn hơn $3$ là $ t_0 = 4$ ; Giá trị  $ t_0 = 4$ sẽ tương ứng với $ c =2$.

 

Suy ra : $ g^{'} (c) = 0  \Leftrightarrow c = 2$.

 

Mặt khác, dễ thấy $ g^{'}(c)$ là hàm đơn điệu tăng trên $ (0; + \infty )$ và $ g^{'}(2) =0$.  Suy ra $ g^{'}(c) \leq 0$ với mọi $ c \in  \left( 0; \  2 \right]$ và $ g^{'}(c) \geq 0$ với mọi $ c \in \left[2; \ + \infty \right)$. Nên hiển nhiên hàm $g(c)$ là hàm số liên tục trên $\mathbb{R}^{+}$, đơn điệu giảm trên  $ \left( 0; \  2 \right]$ và đơn điệu tăng trên $ \left[2; \ + \infty \right)$. Từ đây suy ra: $ g(c) \geq g(2) = \frac{15}{2}$  $(4)$ .

 

Từ $ (1); \ (2); \ (3) ; \ (4)$ Ta suy ra: $ a + b + c \geq \frac{15}{2}$

 

Dấu đẳng thức xảy ra khi và chỉ khi $ a = 3; \ b = \frac{5}{2} ;\  c= 2$.

 

Bài toán theo đó được giải quyết hoàn toàn  . Bài Toán Hay. Lâu Lâu mới có $1$ bài bất đẳng thức hay và sáng tạo thế này :)




#731387 $n_{min}$ thỏa tất hệ số $P(x)=(x^2-3x+3)(x+1)^n...

Đã gửi bởi supermember on 30-10-2021 - 22:08 trong Đa thức

Hy vọng là giải đúng  :D

Bài này đề đúng phải phát biểu là: Chứng minh rằng kể từ một chỉ số nguyên dương $n$ nào đó thì mọi hệ số của đa thức $(x^2-3x+3)(x+1)^n$ đều là số dương. Tìm giá trị nhỏ nhất có thể có của  chỉ số nguyên dương $n$ này.

 

 

Ta thử $1$ vài giá trị $n$ đầu tiên để tạo đà:

 

* $n=1$ không thỏa mãn đề bài do hệ số của $x^2$ trong khai triển $ (x^2 - 3x+3)(x+1)$ là $ -2 <0$

 

* $ n =2$ không thỏa mãn đề bài do hệ số của $x^3$ trong khai triển $ (x^2 - 3x+3)(x^2+ 2x+1)$ là $ -1 <0$

 

* $ n = 3$ không thỏa mãn đề bài do hệ số của $ x^4 $ trong khai triển  $ (x^2 - 3x+3)(x^3+ 3x^2 +3x+1)$ là $0$
Vậy rõ ràng là $n \ge 4$

 

Khi $n \geq 4$ thì do $P(x)$ là đa thức bậc $n+2$ và: $ [x^{n+2}] P(x) = 1 >0$, $  [x^{n+1}] P(x) = n-3 >0$, $ [x^1] P(x) = 3n-3 >0$, $ [x^0] P(x) = 3 >0$

 

Nên ta chỉ cần xét các hệ số của $x^k$ với $ 2 \leq k \leq n$

 

$ [x^{k}] P(x) = \binom{n}{k-2} - 3 \binom{n}{k-1} + 3 \binom{n}{k}$

 

$ = \frac{n!}{ (k-2)! \cdot (n-k+2)!}  - \frac{3n!}{ (k-1)! \cdot (n-k+1)!} + \frac{3n!}{ k! \cdot (n-k)!}  $

 

$ = \frac{n!}{ k! \cdot (n-k+2)!}  \cdot \left( k(k-1) -3k(n-k+2) + 3(n-k+1)(n-k+2) \right)$ 

 

$ = \frac{n!}{ k! \cdot (n-k+2)!}  \cdot \left( 7k^2 -(9n+16)k + (3n^2+9n+6) \right)$ 

 

Xét tam thức bậc $2$  : $Q(k) = 7k^2 -(9n+16)k + (3n^2+9n+6)$

 

Ta có $ \Delta = (9n+16)^2 - 28(3n^2+9n+6) = -3n^2 + 36n+ 88 = -3(n-6)^2 + 196 < 0$ với mọi $n \geq 15$

 

Mà $Q(k)$ là tam thức bậc $2$ ẩn $k$ có hệ số của $k^2$ là $7>0$ , suy ra nếu $n \geq 15$ thì $Q(k) >0$ với mọi $k \in \mathbb{R}$

Từ đây suy ra với $n \geq 15$ thì  mọi hệ số của $ P(x)$ đều là số nguyên dương.

 

Do đó, khẳng định thứ nhất của bài toán đã được chứng minh.

 

Để tìm giá trị $n$ nhỏ nhất thỏa bài toán thì ta chỉ cần thử trực tiếp từ $4$ đến $14$. Quá trình này không khó, tạm thời đi ngủ để tối nay chờ MU xuống hạng.

 

Tiếp tục: MU thắng nhưng sau mùa này nhiều khả năng vẫn xuống giải hạng nhất.
 
* $ n = 4$ không thỏa mãn đề bài do hệ số của $ x^3 $ trong khai triển  $ (x^2 - 3x+3)(x^4+ 4x^3+6x^2 +4x+1)$ là $4- 18+ 12 = -2 < 0$

 

* $ n = 5$ không thỏa mãn đề bài do hệ số của $ x^4 $ trong khai triển  $ (x^2 - 3x+3)(x+1)^5$ là $ \binom{5}{2} - 3 \binom{5}{3} + 3 \binom{5}{4} = -5 <0$

 

* $ n = 6$ không thỏa mãn đề bài do hệ số của $ x^5 $ trong khai triển  $ (x^2 - 3x+3)(x+1)^6$ là $ \binom{6}{3} - 3 \binom{6}{4} + 3 \binom{6}{5} = -7 <0$

 

* $ n = 7$ không thỏa mãn đề bài do hệ số của $ x^6 $ trong khai triển  $ (x^2 - 3x+3)(x+1)^7$ là $ \binom{7}{4} - 3 \binom{7}{5} + 3 \binom{7}{6} = -7 <0$

 

* $ n = 8$ không thỏa mãn đề bài do hệ số của $ x^7 $ trong khai triển  $ (x^2 - 3x+3)(x+1)^8$ là $ \binom{8}{5} - 3 \binom{8}{6} + 3 \binom{8}{7} = -4 <0$

 

* $ n = 9$ không thỏa mãn đề bài do hệ số của $ x^7 $ trong khai triển  $ (x^2 - 3x+3)(x+1)^9$ là $ \binom{9}{5} - 3 \binom{9}{6} + 3 \binom{9}{7} = -18 <0$

 

* $ n = 10$ không thỏa mãn đề bài do hệ số của $ x^8 $ trong khai triển  $ (x^2 - 3x+3)(x+1)^{10}$ là $ \binom{10}{6} - 3 \binom{10}{7} + 3 \binom{10}{8} = -15 <0$

 

* $ n = 11$ không thỏa mãn đề bài do hệ số của $ x^8 $ trong khai triển  $ (x^2 - 3x+3)(x+1)^{11}$ là $ \binom{11}{6} - 3 \binom{11}{7} + 3 \binom{11}{8} = -33 <0$

 

* $n = 12$ không thỏa mãn đề bài do hệ số của $ x^9 $ trong khai triển  $ (x^2 - 3x+3)(x+1)^{12}$ là $ \binom{12}{7} - 3 \binom{12}{8} + 3 \binom{12}{9} = -33 <0$

 

* $ n = 13$ không thỏa mãn đề bài do hệ số của $ x^9 $ trong khai triển  $ (x^2 - 3x+3)(x+1)^{13}$ là $ \binom{13}{7} - 3 \binom{13}{8} + 3 \binom{13}{9} = 0$

 

* $ n = 14$ không thỏa mãn đề bài do hệ số của $ x^{10} $ trong khai triển  $ (x^2 - 3x+3)(x+1)^{14}$ là $ \binom{14}{8} - 3 \binom{14}{9} + 3 \binom{14}{10} = 0$

 

Do đó $ n_{ \min} = 15$ và bài toán theo đó được giải quyết hoàn toàn.




#731493 $3(x^2-x+1)(y^2-y+1)(z^2-z+1)\geq x^2y^2z^2+xyz+1 (\forall x,y...

Đã gửi bởi supermember on 06-11-2021 - 12:28 trong Bất đẳng thức - Cực trị

Bài này có lời giải khác đơn giản & tự nhiên hơn.

Chờ chút đi ăn cơm. Lát post lên.

 

Bắt đầu: Hi vọng là giải đúng  :D 
 

Xét tam thức bậc $2$  : $ g(x) = x^2 - x +1$

 

Ta thấy rằng bất đẳng thức cần chứng minh tương đương với: $ 3(x^2 - x+1) g(y) g(z) \geq x^2 y^2 z^2 + xyz +1$

 

Tức là ta cần đi chứng minh $  d(x) = (3g(y)g(z) - y^2 z^2 ) x^2 - (3g(y)g(z) +yz)x + (3g(y)g(z) -1) \geq 0$

 

Vế trái của bất đẳng thức này là tam thức bậc $2$ ẩn $x$, một cách tự nhiên, ta xét $ \Delta$ của tam thức này:

 

$  \Delta = (3g(y)g(z) +yz)^2 - 4 \left( 3g(y)g(z) - y^2 z^2 \right) \cdot  \left( 3g(y)g(z) -1 \right)$

 

$ = 9 g^2 (y) g^2 (z) + 6yzg(y)g(z) + y^2 z^2 - 36 g^2 (y) g^2 (z) + 12 g(y)g(z) + 12 y^2 z^2 g(y)g(z) - 4y^2 z^2 $

 

$ = -27g^2 (y) g^2 (z) + 6yzg(y)g(z) - 3y^2 z^2 + 12 g(y)g(z) + 12 y^2 z^2 g(y)g(z)$

 

$ \implies  \Delta = -3 \left( g(y)g(z) - yz \right) ^2 - 12 g(y)g(z) \left( 2g(y)g(z) -  y^2 z^2 -1 \right)$   $ ( \bigstar )$

 

Mục đích sau cùng của ta là đi chứng minh $ \Delta \leq 0$ và để chứng minh điều này, ta sẽ đi chứng minh:

 

$ 2g(y)g(z) -  y^2 z^2 -1 \geq 0$   $ ( \bigstar   \bigstar)$

 

Thật vậy, nếu ta tiếp tục sử dụng kỹ thuật như từng dùng ở trên, thì ta dễ thấy là $ ( \bigstar   \bigstar)$ tương đương với:

 

$  h(y) = (z^2 - 2z+2)y^2 - 2(z^2-z+1)y + (2z^2 - 2z +1) \geq 0$

 

Tiếp tục xét $ \Delta^{'}_{h}$ thì ta có:

 

$ \Delta^{'}_{h} = (z^2-z+1)^2 - (z^2 - 2z+2) \cdot (2z^2 - 2z +1) = z^4 + z^2 +1 - 2z^3 -2z+ 2z^2 - 2z^4 + 2z^3 - z^2 + 4z^3 - 4z^2 + 2z - 4z^2 + 4z -2$

 

$ = -z^4 + 4z^3 - 6z^2 + 4z -1 = -(z-1)^4 \leq 0$ với mọi $ z \in \mathbb{R}$

 

Ta có: $h(y)$ là tam thức bậc $2$ có hệ số của $y^2$ là $ z^2 - 2z+2 = (z-1)^2 +1 >0$ và định thức $ \Delta^{'}_{h} \leq 0$ với mọi $ z \in \mathbb{R}$

 

Suy ra $ h(y) \geq 0$ với mọi  $ y \in \mathbb{R}$

 

Suy ra  $ ( \bigstar   \bigstar)$ được chứng minh hoàn toàn.

 

Từ $    ( \bigstar ); ( \bigstar   \bigstar)$ ta suy ra:  $  \Delta \leq 0$ với mọi $y;z \in \mathbb{R}$

 

Suy ra $  d(x)$ là tam thức bậc $2$ có hệ số $x^2$ là  $3g(y)g(z) - y^2 z^2 > 2g(y)g(z) -  y^2 z^2 -1  \geq 0$ và định thức $  \Delta \leq 0$ với mọi $y;z \in \mathbb{R}$

 

Nên theo kiến thức cơ bản về tam thức bậc $2$ thì hiển nhiên $ d(x) \geq 0$ với mọi $ x \in \mathbb{R}$

 

Và bài toán theo đó được giải quyết hoàn toàn. Đẳng thức xảy ra khi và chỉ khi : $ x=y=z=1$

 

Và ta còn thu được kết quả mạnh hơn nhiều:

 

Bất đẳng thức đã cho vẫn đúng khi $x;y;z \in \mathbb{R}$ và điều kiện đã cho là thừa . :)




#732125 $\frac{2}{1+a^2}-\frac{2}{1+b^2}+\frac{3}{1+c^2} \le...

Đã gửi bởi supermember on 18-12-2021 - 19:13 trong Bất đẳng thức - Cực trị

Liệu rằng có thể sử dụng phương pháp đã trình bày trong bài BĐT ở TOPIC https://diendantoanh...lâm-đồng-2022/ để giải quyết bài toán này không?

Supermember nghĩ rằng làm được, các bạn hãy thử xem :)

 

Ok, let's go!. Hi vọng là giải đúng!

Từ giả thiết bài toán ta suy ra: $ a = \frac{b-c}{1+bc}$

 

Do giả thiết bài toán $a;b ; c$ là những số thực dương nên hiển nhiên phải có: $b >c$

 

Do đó $ \mathcal{P} (a;b;c) = \frac{2}{1+a^2} - \frac{2}{1+b^2} + \frac{3}{1+c^2} = \frac{2}{1+ \frac{(b-c)^2}{(1+bc)^2}} - \frac{2}{1+b^2} + \frac{3}{1+c^2} $

 

$=  \frac{2 (1+bc)^2}{(1+b^2)(1+c^2)}- \frac{2}{1+b^2} + \frac{3}{1+c^2} =  \frac{2}{1+c^2}  \cdot \frac{ (1+bc)^2 - (1+c^2)}{1+b^2}  + \frac{3}{1+c^2}$     $(1)$

 

Với mỗi giá trị $c$ cố định, Ta sẽ đi khảo sát hàm số : $ f(x) =  \frac{ (1+xc)^2 - (1+c^2)}{1+x^2} = \frac{ 2cx+  c^2 x^2 - c^2}{1+x^2} $ trên $ (c; +\infty)$

 

Thật vậy, $ f^{'}(x) =   \frac{ (2c^2 x+2c)(1+x^2) - 2x(2cx+  c^2 x^2 - c^2) }{(1+x^2)^2}  =   \frac{ 2c(1 +2cx -x^2) }{(1+x^2)^2}$

 

Do đó $ f^{'}(x) =0 \Leftrightarrow 1 + 2cx - x^2 = 0  \Leftrightarrow (x-c)^2 = c^2 +1 \Leftrightarrow x = x_0= \sqrt{c^2+1} +c$

 

Từ kiến thức cơ bản về dấu của tam thức bậc $2$, Ta dễ thấy rằng $f(x)$ đơn điệu tăng trên $(c; x_0 ]$ và đơn điệu giảm trên $ [x_0 ; + \infty)$ do đó:

 

$ f(x) \leq f(x_0)$ với mọi $x$ thuộc $  (c; +\infty)$

 

Xét khai triển $f(x_0)$ thì bằng tính toán và rút gọn, ta có: $ f(x_0) = c^2 + \frac{c}{c + \sqrt{1+c^2}}$

 

Suy ra $ \mathcal{P} (a;b;c) \leq \frac{2}{1+c^2}  \cdot \left( c^2 + \frac{c}{c + \sqrt{1+c^2}} \right)   + \frac{3}{1+c^2} = g(c)$      $(2)$

 

Đoạn kết của bài toán chỉ còn là đi khảo sát hàm $ g(c)$ này trên $ (0; \infty)$ mà thôi. Tuy rằng không phải dễ nhưng cũng sẽ không quá khó :)

 

Ta dùng kỹ năng cơ bản về các " lượng liên hợp", thật vậy:

$ g(c) = \frac{2}{1+c^2}  \cdot \left( c^2 + \frac{c \cdot (  \sqrt{1+c^2} -c) }{(c + \sqrt{1+c^2}) \cdot (  \sqrt{1+c^2} -c) } \right)   + \frac{3}{1+c^2} $

 

$ = \frac{2}{1+c^2}  \cdot \left( c^2 + c \cdot (  \sqrt{1+c^2} -c ) \right)  + \frac{3}{1+c^2} $

 

$ \implies g(c) = \frac{2c}{ \sqrt{1+c^2}}  + \frac{3}{1+c^2} $      $(3)$

 

Đặt ẩn phụ $ t =  \sqrt{1+c^2}$ và rõ ràng $ t> 1$

 

$g(c) = \frac{ 2 \sqrt{t^2-1}}{t} + \frac{3}{t^2} = h(t)$ . Ta khảo sát hàm $h(t)$ trên $(1; + \infty)$

 

Ta có $ h^{'}(t) = \frac{2t - 6 \sqrt{t^2-1}}{t^3 \sqrt{t^2-1}} =  \frac{2(9 - 8t^2)}{ t^3 \cdot \sqrt{t^2-1} \cdot ( t+ 3 \sqrt{t^2-1} )} $ Thì $ h^{'}(t) = 0  \Leftrightarrow  9-8t^2 =0 \Leftrightarrow  8t^2 =9 \Leftrightarrow t = t_0 = \frac{3}{ 2 \sqrt{2}} $

 

Ta dễ thấy $ h(t)$ là hàm đồng biến trên $ (1; t_0 ]$ và là hàm nghịch biến trên $ [t_0 ; +\infty)$

 

Do đó $ g(c) = h(t) \leq h(t_0) = \frac{10}{3} $       $(4)$

 

Do đó , từ $(1); \ (2); \ (3); \ (4)$, ta suy ra:

 

Giá trị lớn nhất của $  \mathcal{P} (a;b;c)$ là $ \frac{10}{3} $, đạt được khi $ c = \frac{1}{ 2 \sqrt{2}} ; b = \sqrt{2} ;  a =  \frac{1}{ \sqrt{2}}$

 

Bài Toán Theo đó được giải quyết hoàn toàn. Và thực ra ta còn giải được bài toán ở dạng khó hơn là tìm GTLN thay vì chỉ đi chứng minh bất đẳng thức.